CONNEXION
  • RetourJeux
    • Tests
    • Soluces
    • Previews
    • Sorties
    • Hit Parade
    • Les + attendus
    • Tous les Jeux
  • RetourActu
    • Culture Geek
    • Astuces
    • Réalité Virtuelle
    • Rétrogaming
    • Toutes les actus
  • RetourHigh-Tech
    • Actus JVTECH
    • Bons plans
    • Tutoriels
    • Tests produits High-Tech
    • Guides d'achat High-Tech
    • JVTECH
  • RetourVidéos
    • A la une
    • Gaming Live
    • Vidéos Tests
    • Vidéos Previews
    • Gameplay
    • Trailers
    • Chroniques
    • Replay Web TV
    • Toutes les vidéos
  • RetourForums
    • Hardware PC
    • PS5
    • Switch
    • Xbox Series
    • Overwatch 2
    • FUT 23
    • League of Legends
    • Genshin Impact
    • Tous les Forums
  • PC
  • PS5
  • Xbox Series
  • PS4
  • One
  • Switch
  • Wii U
  • iOS
  • Android
  • MMO
  • RPG
  • FPS
En ce moment Genshin Impact Valhalla Breath of the wild Animal Crossing GTA 5 Red dead 2
Etoile Abonnement RSS

Sujet : Carrés dans un carré

DébutPage précedente
1
Page suivantePage suivante
Amandin Amandin
MP
Niveau 10
01 mars 2015 à 15:58:47

Salut,

j'ai trouvé ce problème dans un bouquin ("problem solving stragegy" de Engel) sans solution, ça à l'air tout con mais je ne vois pas de preuve simple :

Dans un carré se trouvent deux carrés qui ne se croisent pas. Montrer que la somme de leur côté ne peut pas dépasser celui du contenant.

Des idées?

Amandin Amandin
MP
Niveau 10
01 mars 2015 à 16:23:43

"Comme ils ne se croisent pas et qu'ils faut qu'ils soient disposés ainsi pour être le plus grand possible,"

Comment tu prouves ça? Ca ne me semble pas évident du tout!

Pseudo supprimé
Niveau 10
01 mars 2015 à 16:25:16

Si je comprends bien, les carrés ne se croisent pas, mais en plus de ça aucun des deux carrés ne peut être à l'intérieur de l'autre ? :(

Amandin Amandin
MP
Niveau 10
01 mars 2015 à 16:27:59

Oui les deux carrés ne se croisent pas et ne sont pas imbriqués, sinon c'est trivialement faux

Amandin Amandin
MP
Niveau 10
01 mars 2015 à 16:40:26

Ce que tu conjectures c'est ce qu'on veut prouver!

Amandin Amandin
MP
Niveau 10
01 mars 2015 à 16:44:32

"B et C n'ont pas d'intersection: donc x+y<=z"

C'est ce qu'on veut prouver donc on peut pas l'affirmer comme ça.

baptisteb39 baptisteb39
MP
Niveau 9
01 mars 2015 à 16:46:40

ah oui désolé ma démonstration est incomplète mais ça me semble tellement évident :honte:

Amandin Amandin
MP
Niveau 10
01 mars 2015 à 16:49:55

"Je sais pas mais si x>1-y, les deux carrés vont nécessairement se croiser."

Pourquoi? On tourne un peu en rond en disant que c'est évident ou nécessairement vrai...

Je suis d'accord avec vous que ça paraît évident mais je n'arrive pas à le montrer proprement.

C'est un problème énoncé par Erdös donc je ne pense pas que ce soit si évident que ça vu l'auteur...

Amandin Amandin
MP
Niveau 10
02 mars 2015 à 16:29:15

Up, j'ai posé le problème à un prof aujourd'hui, j'attends sa réponse.

Vistiche Vistiche
MP
Niveau 10
02 mars 2015 à 17:52:45

J'ai fait tout un blabla inutile mais je l'ai supprimé car ça n'apportait rien. :hap:
Je peux le reposter si vous voulez.

Vistiche Vistiche
MP
Niveau 10
02 mars 2015 à 18:11:09

Bon je reposte pour que vous me disiez exactement pourquoi c'est de la merde ce que je dis.

On note le carré ABDC et on se place dans le repère (A;B;C).
On note x le côté du 1er carré et on désigne par le couple (a;b) les coordonnées du sommet en bas à gauche du 1er carré. Avec a et b et x appartenant à [0;1]

L'ensemble des points contenus par le carré on donc pour coordonnées [a+k;b+k] avec k variant de 0 à x.

Le sommet en bas à gauche du 2ème carré (de côté y) a pour coordonnées [c;d] donc l'ensemble des points contenus dans le carré ont pour coordonnées [c+l;d+l] avec l variant de 0 à y.

Déjà il faut que a+x<=1 de même pour b+x, c+y et d+y.
Ensuite on regarde pour quelles valeurs de a,b, c et d on a une intersection nulle lorsque x et y sont fixés.

[a;a+k]inter[c;c+l]=∅ lorsque a+k<c donc comme k varie de 0 à x, a+x<c, l'écart est maximum lorsque a et c sont les plus éloignés donc quand a=0 et c=1
On tient le même raisonnement pour b et d.

On a donc au final la meilleure optimisation lorsque a=0, b=0, c=1 et d=1

Ensuite lorsque les coordonnées sont fixées: une condition sur les longueurs.
Il faut toujours que [a;a+k]inter[c;c+l]=∅ donc comme on a vu que la meilleure optimisation était pour a=0, b=0, c=1 et d=1, on a: [0;x]inter[1-y;1]=∅ On a donc bien x<=1-y

Message édité le 02 mars 2015 à 18:11:42 par Vistiche
Pseudo supprimé
Niveau 10
02 mars 2015 à 18:38:20

Le plus dur c'est de prendre en compte l'orientation des carrés non ? Rien ne suppose que les côtés des carrés soient "parallèles"

Morphisme Morphisme
MP
Niveau 10
02 mars 2015 à 18:41:50

Une réponse élégante au problème :
http://math.stackexchange.com/questions/244474/two-squares-in-a-box

Et pour info Vistiche le problème dans ton raisonnement c'est que tu supposes que les carrés sont placés "droits" comme l'a dit 111010001100111 (et en effet c'est le placement optimal si tu veux maximiser la somme des côtés, mais c'est justement ça qui est difficile à prouver !)

Message édité le 02 mars 2015 à 18:46:21 par Morphisme
Amandin Amandin
MP
Niveau 10
02 mars 2015 à 19:05:54

Morphisme :merci:

effectivement à partir du lemme du triangle de ton lien c'est presque immédiat mais il fallait avoir l'idée de tracer la droite qui sépare les deux carrés...

Bon et maintenant, quid pour 3 carrés dans un carré? :malade:

Vistiche Vistiche
MP
Niveau 10
02 mars 2015 à 19:21:10

Ah oui ok je vois pourquoi ce que je dis est inutile maintenant. :hap:

Vistiche Vistiche
MP
Niveau 10
02 mars 2015 à 20:06:07

Bah pour 3 carrés, il est facile de trouver un cas où la somme de leurs côtés est supérieure à celle du grand carré.:(

DébutPage précedente
1
Page suivantePage suivante
Répondre
Prévisu
?
Victime de harcèlement en ligne : comment réagir ?
Infos 0 connecté(s)

Gestion du forum

Modérateurs : HypoBowling
Contacter les modérateurs - Règles du forum

Sujets à ne pas manquer

La vidéo du moment